How many permutations of a given array result in BST's of height 2? - algorithm

A BST is generated (by successive insertion of nodes) from each permutation of keys from the set {1,2,3,4,5,6,7}. How many permutations determine trees of height two?
I been stuck on this simple question for quite some time. Any hints anyone.
By the way the answer is 80.

Consider how the tree would be height 2?
-It needs to have 4 as root, 2 as the left child, 6 right child, etc.
How come 4 is the root?
-It needs to be the first inserted. So we have one number now, 6 still can move around in the permutation.
And?
-After the first insert there are still 6 places left, 3 for the left and 3 for the right subtrees. That's 6 choose 3 = 20 choices.
Now what?
-For the left and right subtrees, their roots need to be inserted first, then the children's order does not affect the tree - 2, 1, 3 and 2, 3, 1 gives the same tree. That's 2 for each subtree, and 2 * 2 = 4 for the left and right subtrees.
So?
In conclusion: C(6, 3) * 2 * 2 = 20 * 2 * 2 = 80.

Note that there is only one possible shape for this tree - it has to be perfectly balanced. It therefore has to be this tree:
4
/ \
2 6
/ \ / \
1 3 5 7
This requires 4 to be inserted first. After that, the insertions need to build up the subtrees holding 1, 2, 3 and 5, 6, 7 in the proper order. This means that we will need to insert 2 before 1 and 3 and need to insert 6 before 5 and 7. It doesn't matter what relative order we insert 1 and 3 in, as long as they're after the 2, and similarly it doesn't matter what relative order we put 5 and 7 in as long as they're after 6. You can therefore think of what we need to insert as 2 X X and 6 Y Y, where the X's are the children of 2 and the Y's are the children of 6. We can then find all possible ways to get back the above tree by finding all interleaves of the sequences 2 X X and 6 Y Y, then multiplying by four (the number of ways of assigning X and Y the values 1, 3, 5, and 7).
So how many ways are there to interleave? Well, you can think of this as the number of ways to permute the sequence L L L R R R, since each permutation of L L L R R R tells us how to choose from either the Left sequence or the Right sequence. There are 6! / 3! 3! = 20 ways to do this. Since each of those twenty interleaves gives four possible insertion sequences, there end up being a total of 20 × 4 = 80 possible ways to do this.
Hope this helps!

I've created a table for the number of permutations possible with 1 - 12 elements, with heights up to 12, and included the per-root break down for anybody trying to check that their manual process (described in other answers) is matching with the actual values.
http://www.asmatteringofit.com/blog/2014/6/14/permutations-of-a-binary-search-tree-of-height-x

Here is a C++ code aiding the accepted answer, here I haven't shown the obvious ncr(i,j) function, hope someone will find it useful.
int solve(int n, int h) {
if (n <= 1)
return (h == 0);
int ans = 0;
for (int i = 0; i < n; i++) {
int res = 0;
for (int j = 0; j < h - 1; j++) {
res = res + solve(i, j) * solve(n - i - 1, h - 1);
res = res + solve(n - i - 1, j) * solve(i, h - 1);
}
res = res + solve(i, h - 1) * solve(n - i - 1, h - 1);
ans = ans + ncr(n - 1, i) * res;
}
return ans
}

The tree must have 4 as the root and 2 and 6 as the left and right child, respectively. There is only one choice for the root and the insertion should start with 4, however, once we insert the root, there are many insertion orders. There are 2 choices for, the second insertion 2 or 6. If we choose 2 for the second insertion, we have three cases to choose 6: choose 6 for the third insertion, 4, 2, 6, -, -, -, - there are 4!=24 choices for the rest of the insertions; fix 6 for the fourth insertion, 4, 2, -, 6, -,-,- there are 2 choices for the third insertion, 1 or 3, and 3! choices for the rest, so 2*3!=12, and the last case is to fix 6 in the fifth insertion, 4, 2, -, -, 6, -, - there are 2 choices for the third and fourth insertion ((1 and 3), or (3 and 1)) as well as for the last two insertions ((5 and 7) or (7 and 5)), so there are 4 choices. In total, if 2 is the second insertion we have 24+12+4=40 choices for the rest of the insertions. Similarly, there are 40 choices if the second insertion is 6, so the total number of different insertion orders is 80.

Related

Find greater number form self in left side and smaller number form self in right side

Consider an array a of n integers, indexed from 1 to n.
For every index i such that 1<i<n, define:
count_left(i) = number of indices j such that 1 <= j < i and a[j] > a[i];
count_right(i) = number of indices j such that i < j <= n and a[j] < a[i];
diff(i) = abs(count_left(i) - count_right(i)).
The problem is: given array a, find the maximum possible value of diff(i) for 1 < i < n.
I got solution by brute force. Can anyone give better solution?
Constraint: 3 < n <= 10^5
Example
Input Array: [3, 6, 9, 5, 4, 8, 2]
Output: 4
Explanation:
diff(2) = abs(0 - 3) = 3
diff(3) = abs(0 - 4) = 4
diff(4) = abs(2 - 2) = 0
diff(5) = abs(3 - 1) = 2
diff(6) = abs(1 - 1) = 0
maximum is 4.
O(nlogn) approach:
Walk through array left to right and add every element to augmented binary search tree (RB, AVL etc) containing fields of subtree size, initial index and temporary rank field. So immediately after adding we know rank of element in the current tree state.
lb = index - temprank
is number of left bigger elements - remember it in temprank field.
After filling the tree with all items traverse tree again, retrieving final element rank.
rs = finalrank - temprank
is number of right smaller elements. Now just get abs of difference of lb and rs
diff = abs(lb - rs) = abs(index - temprank - finalrank + temprank ) =
abs(index - finalrank)
But ... we can see that we don't need temprank at all.
Moreover - we don't need binary tree!
Just perform sorting of pairs (element; initial index) by element key and get max absolute difference of new_index - old_index (except for old indices 1 and n)
a 3, 6, 9, 5, 4, 8, 2
old 2 3 4 5 6
new 5 7 4 3 6
dif 3 4 0 2 0
Python code for concept checking
a = [3, 6, 9, 5, 4, 8, 2]
b = sorted([[e,i] for i,e in enumerate(a)])
print(b)
print(max([abs(n-o[1]) if 0<o[1]<len(a)-1 else 0 for n,o in enumerate(b)]))

Kth element in transformed array

I came across this question in recent interview :
Given an array A of length N, we are supposed to answer Q queries. Query form is as follows :
Given x and k, we need to make another array B of same length such that B[i] = A[i] ^ x where ^ is XOR operator. Sort an array B in descending order and return B[k].
Input format :
First line contains interger N
Second line contains N integers denoting array A
Third line contains Q i.e. number of queries
Next Q lines contains space-separated integers x and k
Output format :
Print respective B[k] value each on new line for Q queries.
e.g.
for input :
5
1 2 3 4 5
2
2 3
0 1
output will be :
3
5
For first query,
A = [1, 2, 3, 4, 5]
For query x = 2 and k = 3, B = [1^2, 2^2, 3^2, 4^2, 5^2] = [3, 0, 1, 6, 7]. Sorting in descending order B = [7, 6, 3, 1, 0]. So, B[3] = 3.
For second query,
A and B will be same as x = 0. So, B[1] = 5
I have no idea how to solve such problems. Thanks in advance.
This is solvable in O(N + Q). For simplicity I assume you are dealing with positive or unsigned values only, but you can probably adjust this algorithm also for negative numbers.
First you build a binary tree. The left edge stands for a bit that is 0, the right edge for a bit that is 1. In each node you store how many numbers are in this bucket. This can be done in O(N), because the number of bits is constant.
Because this is a little bit hard to explain, I'm going to show how the tree looks like for 3-bit numbers [0, 1, 4, 5, 7] i.e. [000, 001, 100, 101, 111]
*
/ \
2 3 2 numbers have first bit 0 and 3 numbers first bit 1
/ \ / \
2 0 2 1 of the 2 numbers with first bit 0, have 2 numbers 2nd bit 0, ...
/ \ / \ / \
1 1 1 1 0 1 of the 2 numbers with 1st and 2nd bit 0, has 1 number 3rd bit 0, ...
To answer a single query you go down the tree by using the bits of x. At each node you have 4 possibilities, looking at bit b of x and building answer a, which is initially 0:
b = 0 and k < the value stored in the left child of the current node (the 0-bit branch): current node becomes left child, a = 2 * a (shifting left by 1)
b = 0 and k >= the value stored in the left child: current node becomes right child, k = k - value of left child, a = 2 * a + 1
b = 1 and k < the value stored in the right child (the 1-bit branch, because of the xor operation everything is flipped): current node becomes right child, a = 2 * a
b = 1 and k >= the value stored in the right child: current node becomes left child, k = k - value of right child, a = 2 * a + 1
This is O(1), again because the number of bits is constant. Therefore the overall complexity is O(N + Q).
Example: [0, 1, 4, 5, 7] i.e. [000, 001, 100, 101, 111], k = 3, x = 3 i.e. 011
First bit is 0 and k >= 2, therefore we go right, k = k - 2 = 3 - 2 = 1 and a = 2 * a + 1 = 2 * 0 + 1 = 1.
Second bit is 1 and k >= 1, therefore we go left (inverted because the bit is 1), k = k - 1 = 0, a = 2 * a + 1 = 3
Third bit is 1 and k < 1, so the solution is a = 2 * a + 0 = 6
Control: [000, 001, 100, 101, 111] xor 011 = [011, 010, 111, 110, 100] i.e. [3, 2, 7, 6, 4] and in order [2, 3, 4, 6, 7], so indeed the number at index 3 is 6 and the solution (always talking about 0-based indexing here).

Finding the maximum possible sum/product combination of integers

Given an input of a list of N integers always starting with 1, for example: 1, 4, 2, 3, 5. And some target integer T.
Processing the list in order, the algorithm decides whether to add or multiply the number by the current score to achieve the maximum possible output < T.
For example: [input] 1, 4, 2, 3, 5 T=40
1 + 4 = 5
5 * 2 = 10
10 * 3 = 30
30 + 5 = 35 which is < 40, so valid.
But
1 * 4 = 4
4 * 2 = 8
8 * 3 = 24
24 * 5 = 120 which is > 40, so invalid.
I'm having trouble conceptualizing this in an algorithm -- I'm just looking for advice on how to think about it or at most pseudo-code. How would I go about coding this?
My first instinct was to think about the +/* as 1/0, and then test permutations like 0000 (where length == N-1, I think), then 0001, then 0011, then 0111, then 1111, then 1000, etc. etc.
But I don't know how to put that into pseudo-code given a general N integers. Any help would be appreciated.
You can use recursive to implement the permutations. Python code below:
MINIMUM = -2147483648
def solve(input, T, index, temp):
# if negative value exists in input, remove below two lines
if temp >= T:
return MINIMUM
if index == len(input):
return temp
ans0 = solve(input, T, index + 1, temp + input[index])
ans1 = solve(input, T, index + 1, temp * input[index])
return max(ans0, ans1)
print(solve([1, 4, 2, 3, 5], 40, 1, 1))
But this method requires O(2^n) time complexity.

Google Interview: Arrangement of Blocks

You are given N blocks of height 1…N. In how many ways can you arrange these blocks in a row such that when viewed from left you see only L blocks (rest are hidden by taller blocks) and when seen from right you see only R blocks? Example given N=3, L=2, R=1 there is only one arrangement {2, 1, 3} while for N=3, L=2, R=2 there are two ways {1, 3, 2} and {2, 3, 1}.
How should we solve this problem by programming? Any efficient ways?
This is a counting problem, not a construction problem, so we can approach it using recursion. Since the problem has two natural parts, looking from the left and looking from the right, break it up and solve for just one part first.
Let b(N, L, R) be the number of solutions, and let f(N, L) be the number of arrangements of N blocks so that L are visible from the left. First think about f because it's easier.
APPROACH 1
Let's get the initial conditions and then go for recursion. If all are to be visible, then they must be ordered increasingly, so
f(N, N) = 1
If there are suppose to be more visible blocks than available blocks, then nothing we can do, so
f(N, M) = 0 if N < M
If only one block should be visible, then put the largest first and then the others can follow in any order, so
f(N,1) = (N-1)!
Finally, for the recursion, think about the position of the tallest block, say N is in the kth spot from the left. Then choose the blocks to come before it in (N-1 choose k-1) ways, arrange those blocks so that exactly L-1 are visible from the left, and order the N-k blocks behind N it in any you like, giving:
f(N, L) = sum_{1<=k<=N} (N-1 choose k-1) * f(k-1, L-1) * (N-k)!
In fact, since f(x-1,L-1) = 0 for x<L, we may as well start k at L instead of 1:
f(N, L) = sum_{L<=k<=N} (N-1 choose k-1) * f(k-1, L-1) * (N-k)!
Right, so now that the easier bit is understood, let's use f to solve for the harder bit b. Again, use recursion based on the position of the tallest block, again say N is in position k from the left. As before, choose the blocks before it in N-1 choose k-1 ways, but now think about each side of that block separately. For the k-1 blocks left of N, make sure that exactly L-1 of them are visible. For the N-k blocks right of N, make sure that R-1 are visible and then reverse the order you would get from f. Therefore the answer is:
b(N,L,R) = sum_{1<=k<=N} (N-1 choose k-1) * f(k-1, L-1) * f(N-k, R-1)
where f is completely worked out above. Again, many terms will be zero, so we only want to take k such that k-1 >= L-1 and N-k >= R-1 to get
b(N,L,R) = sum_{L <= k <= N-R+1} (N-1 choose k-1) * f(k-1, L-1) * f(N-k, R-1)
APPROACH 2
I thought about this problem again and found a somewhat nicer approach that avoids the summation.
If you work the problem the opposite way, that is think of adding the smallest block instead of the largest block, then the recurrence for f becomes much simpler. In this case, with the same initial conditions, the recurrence is
f(N,L) = f(N-1,L-1) + (N-1) * f(N-1,L)
where the first term, f(N-1,L-1), comes from placing the smallest block in the leftmost position, thereby adding one more visible block (hence L decreases to L-1), and the second term, (N-1) * f(N-1,L), accounts for putting the smallest block in any of the N-1 non-front positions, in which case it is not visible (hence L stays fixed).
This recursion has the advantage of always decreasing N, though it makes it more difficult to see some formulas, for example f(N,N-1) = (N choose 2). This formula is fairly easy to show from the previous formula, though I'm not certain how to derive it nicely from this simpler recurrence.
Now, to get back to the original problem and solve for b, we can also take a different approach. Instead of the summation before, think of the visible blocks as coming in packets, so that if a block is visible from the left, then its packet consists of all blocks right of it and in front of the next block visible from the left, and similarly if a block is visible from the right then its packet contains all blocks left of it until the next block visible from the right. Do this for all but the tallest block. This makes for L+R packets. Given the packets, you can move one from the left side to the right side simply by reversing the order of the blocks. Therefore the general case b(N,L,R) actually reduces to solving the case b(N,L,1) = f(N,L) and then choosing which of the packets to put on the left and which on the right. Therefore we have
b(N,L,R) = (L+R choose L) * f(N,L+R)
Again, this reformulation has some advantages over the previous version. Putting these latter two formulas together, it's much easier to see the complexity of the overall problem. However, I still prefer the first approach for constructing solutions, though perhaps others will disagree. All in all it just goes to show there's more than one good way to approach the problem.
What's with the Stirling numbers?
As Jason points out, the f(N,L) numbers are precisely the (unsigned) Stirling numbers of the first kind. One can see this immediately from the recursive formulas for each. However, it's always nice to be able to see it directly, so here goes.
The (unsigned) Stirling numbers of the First Kind, denoted S(N,L) count the number of permutations of N into L cycles. Given a permutation written in cycle notation, we write the permutation in canonical form by beginning the cycle with the largest number in that cycle and then ordering the cycles increasingly by the first number of the cycle. For example, the permutation
(2 6) (5 1 4) (3 7)
would be written in canonical form as
(5 1 4) (6 2) (7 3)
Now drop the parentheses and notice that if these are the heights of the blocks, then the number of visible blocks from the left is exactly the number of cycles! This is because the first number of each cycle blocks all other numbers in the cycle, and the first number of each successive cycle is visible behind the previous cycle. Hence this problem is really just a sneaky way to ask you to find a formula for Stirling numbers.
well, just as an empirical solution for small N:
blocks.py:
import itertools
from collections import defaultdict
def countPermutation(p):
n = 0
max = 0
for block in p:
if block > max:
n += 1
max = block
return n
def countBlocks(n):
count = defaultdict(int)
for p in itertools.permutations(range(1,n+1)):
fwd = countPermutation(p)
rev = countPermutation(reversed(p))
count[(fwd,rev)] += 1
return count
def printCount(count, n, places):
for i in range(1,n+1):
for j in range(1,n+1):
c = count[(i,j)]
if c > 0:
print "%*d" % (places, count[(i,j)]),
else:
print " " * places ,
print
def countAndPrint(nmax, places):
for n in range(1,nmax+1):
printCount(countBlocks(n), n, places)
print
and sample output:
blocks.countAndPrint(10)
1
1
1
1 1
1 2
1
2 3 1
2 6 3
3 3
1
6 11 6 1
6 22 18 4
11 18 6
6 4
1
24 50 35 10 1
24 100 105 40 5
50 105 60 10
35 40 10
10 5
1
120 274 225 85 15 1
120 548 675 340 75 6
274 675 510 150 15
225 340 150 20
85 75 15
15 6
1
720 1764 1624 735 175 21 1
720 3528 4872 2940 875 126 7
1764 4872 4410 1750 315 21
1624 2940 1750 420 35
735 875 315 35
175 126 21
21 7
1
5040 13068 13132 6769 1960 322 28 1
5040 26136 39396 27076 9800 1932 196 8
13068 39396 40614 19600 4830 588 28
13132 27076 19600 6440 980 56
6769 9800 4830 980 70
1960 1932 588 56
322 196 28
28 8
1
40320 109584 118124 67284 22449 4536 546 36 1
40320 219168 354372 269136 112245 27216 3822 288 9
109584 354372 403704 224490 68040 11466 1008 36
118124 269136 224490 90720 19110 2016 84
67284 112245 68040 19110 2520 126
22449 27216 11466 2016 126
4536 3822 1008 84
546 288 36
36 9
1
You'll note a few obvious (well, mostly obvious) things from the problem statement:
the total # of permutations is always N!
with the exception of N=1, there is no solution for L,R = (1,1): if a count in one direction is 1, then it implies the tallest block is on that end of the stack, so the count in the other direction has to be >= 2
the situation is symmetric (reverse each permutation and you reverse the L,R count)
if p is a permutation of N-1 blocks and has count (Lp,Rp), then the N permutations of block N inserted in each possible spot can have a count ranging from L = 1 to Lp+1, and R = 1 to Rp + 1.
From the empirical output:
the leftmost column or topmost row (where L = 1 or R = 1) with N blocks is the sum of the
rows/columns with N-1 blocks: i.e. in #PengOne's notation,
b(N,1,R) = sum(b(N-1,k,R-1) for k = 1 to N-R+1
Each diagonal is a row of Pascal's triangle, times a constant factor K for that diagonal -- I can't prove this, but I'm sure someone can -- i.e.:
b(N,L,R) = K * (L+R-2 choose L-1) where K = b(N,1,L+R-1)
So the computational complexity of computing b(N,L,R) is the same as the computational complexity of computing b(N,1,L+R-1) which is the first column (or row) in each triangle.
This observation is probably 95% of the way towards an explicit solution (the other 5% I'm sure involves standard combinatoric identities, I'm not too familiar with those).
A quick check with the Online Encyclopedia of Integer Sequences shows that b(N,1,R) appears to be OEIS sequence A094638:
A094638 Triangle read by rows: T(n,k) =|s(n,n+1-k)|, where s(n,k) are the signed Stirling numbers of the first kind (1<=k<=n; in other words, the unsigned Stirling numbers of the first kind in reverse order).
1, 1, 1, 1, 3, 2, 1, 6, 11, 6, 1, 10, 35, 50, 24, 1, 15, 85, 225, 274, 120, 1, 21, 175, 735, 1624, 1764, 720, 1, 28, 322, 1960, 6769, 13132, 13068, 5040, 1, 36, 546, 4536, 22449, 67284, 118124, 109584, 40320, 1, 45, 870, 9450, 63273, 269325, 723680, 1172700
As far as how to efficiently compute the Stirling numbers of the first kind, I'm not sure; Wikipedia gives an explicit formula but it looks like a nasty sum. This question (computing Stirling #s of the first kind) shows up on MathOverflow and it looks like O(n^2), as PengOne hypothesizes.
Based on #PengOne answer, here is my Javascript implementation:
function g(N, L, R) {
var acc = 0;
for (var k=1; k<=N; k++) {
acc += comb(N-1, k-1) * f(k-1, L-1) * f(N-k, R-1);
}
return acc;
}
function f(N, L) {
if (N==L) return 1;
else if (N<L) return 0;
else {
var acc = 0;
for (var k=1; k<=N; k++) {
acc += comb(N-1, k-1) * f(k-1, L-1) * fact(N-k);
}
return acc;
}
}
function comb(n, k) {
return fact(n) / (fact(k) * fact(n-k));
}
function fact(n) {
var acc = 1;
for (var i=2; i<=n; i++) {
acc *= i;
}
return acc;
}
$("#go").click(function () {
alert(g($("#N").val(), $("#L").val(), $("#R").val()));
});
Here is my construction solution inspired by #PengOne's ideas.
import itertools
def f(blocks, m):
n = len(blocks)
if m > n:
return []
if m < 0:
return []
if n == m:
return [sorted(blocks)]
maximum = max(blocks)
blocks = list(set(blocks) - set([maximum]))
results = []
for k in range(0, n):
for left_set in itertools.combinations(blocks, k):
for left in f(left_set, m - 1):
rights = itertools.permutations(list(set(blocks) - set(left)))
for right in rights:
results.append(list(left) + [maximum] + list(right))
return results
def b(n, l, r):
blocks = range(1, n + 1)
results = []
maximum = max(blocks)
blocks = list(set(blocks) - set([maximum]))
for k in range(0, n):
for left_set in itertools.combinations(blocks, k):
for left in f(left_set, l - 1):
other = list(set(blocks) - set(left))
rights = f(other, r - 1)
for right in rights:
results.append(list(left) + [maximum] + list(right))
return results
# Sample
print b(4, 3, 2) # -> [[1, 2, 4, 3], [1, 3, 4, 2], [2, 3, 4, 1]]
We derive a general solution F(N, L, R) by examining a specific testcase: F(10, 4, 3).
We first consider 10 in the leftmost possible position, the 4th ( _ _ _ 10 _ _ _ _ _ _ ).
Then we find the product of the number of valid sequences in the left and in the right of 10.
Next, we'll consider 10 in the 5th slot, calculate another product and add it to the previous one.
This process will go on until 10 is in the last possible slot, the 8th.
We'll use the variable named pos to keep track of N's position.
Now suppose pos = 6 ( _ _ _ _ _ 10 _ _ _ _ ). In the left of 10, there are 9C5 = (N-1)C(pos-1) sets of numbers to be arranged.
Since only the order of these numbers matters, we could look at 1, 2, 3, 4, 5.
To construct a sequence with these numbers so that 3 = L-1 of them are visible from the left, we can begin by placing 5 in the leftmost possible slot ( _ _ 5 _ _ ) and follow similar steps to what we did before.
So if F were defined recursively, it could be used here.
The only difference now is that the order of numbers in the right of 5 is immaterial.
To resolve this issue, we'll use a signal, INF (infinity), for R to indicate its unimportance.
Turning to the right of 10, there will be 4 = N-pos numbers left.
We first consider 4 in the last possible slot, position 2 = R-1 from the right ( _ _ 4 _ ).
Here what appears in the left of 4 is immaterial.
But counting arrangements of 4 blocks with the mere condition that 2 of them should be visible from the right is no different than counting arrangements of the same blocks with the mere condition that 2 of them should be visible from the left.
ie. instead of counting sequences like 3 1 4 2, one can count sequences like 2 4 1 3
So the number of valid arrangements in the right of 10 is F(4, 2, INF).
Thus the number of arrangements when pos == 6 is 9C5 * F(5, 3, INF) * F(4, 2, INF) = (N-1)C(pos-1) * F(pos-1, L-1, INF)* F(N-pos, R-1, INF).
Similarly, in F(5, 3, INF), 5 will be considered in a succession of slots with L = 2 and so on.
Since the function calls itself with L or R reduced, it must return a value when L = 1, that is F(N, 1, INF) must be a base case.
Now consider the arrangement _ _ _ _ _ 6 7 10 _ _.
The only slot 5 can take is the first, and the following 4 slots may be filled in any manner; thus F(5, 1, INF) = 4!.
Then clearly F(N, 1, INF) = (N-1)!.
Other (trivial) base cases and details could be seen in the C implementation below.
Here is a link for testing the code
#define INF UINT_MAX
long long unsigned fact(unsigned n) { return n ? n * fact(n-1) : 1; }
unsigned C(unsigned n, unsigned k) { return fact(n) / (fact(k) * fact(n-k)); }
unsigned F(unsigned N, unsigned L, unsigned R)
{
unsigned pos, sum = 0;
if(R != INF)
{
if(L == 0 || R == 0 || N < L || N < R) return 0;
if(L == 1) return F(N-1, R-1, INF);
if(R == 1) return F(N-1, L-1, INF);
for(pos = L; pos <= N-R+1; ++pos)
sum += C(N-1, pos-1) * F(pos-1, L-1, INF) * F(N-pos, R-1, INF);
}
else
{
if(L == 1) return fact(N-1);
for(pos = L; pos <= N; ++pos)
sum += C(N-1, pos-1) * F(pos-1, L-1, INF) * fact(N-pos);
}
return sum;
}

How does this work? Weird Towers of Hanoi Solution

I was lost on the internet when I discovered this unusual, iterative solution to the towers of Hanoi:
for (int x = 1; x < (1 << nDisks); x++)
{
FromPole = (x & x-1) % 3;
ToPole = ((x | x-1) + 1) % 3;
moveDisk(FromPole, ToPole);
}
This post also has similar Delphi code in one of the answers.
However, for the life of me, I can't seem to find a good explanation for why this works.
Can anyone help me understand it?
the recursive solution to towers of Hanoi works so that if you want to move N disks from peg A to C, you first move N-1 from A to B, then you move the bottom one to C, and then you move again N-1 disks from B to C. In essence,
hanoi(from, to, spare, N):
hanoi(from, spare, to, N-1)
moveDisk(from, to)
hanoi(spare, to, from, N-1)
Clearly hanoi( _ , _ , _ , 1) takes one move, and hanoi ( _ , _ , _ , k) takes as many moves as 2 * hanoi( _ , _ , _ , k-1) + 1. So the solution length grows in the sequence 1, 3, 7, 15, ... This is the same sequence as (1 << k) - 1, which explains the length of the loop in the algorithm you posted.
If you look at the solutions themselves, for N = 1 you get
FROM TO
; hanoi(0, 2, 1, 1)
0 2 movedisk
For N = 2 you get
FROM TO
; hanoi(0, 2, 1, 2)
; hanoi(0, 1, 2, 1)
0 1 ; movedisk
0 2 ; movedisk
; hanoi(1, 2, 0, 1)
1 2 ; movedisk
And for N = 3 you get
FROM TO
; hanoi(0, 2, 1, 3)
; hanoi(0, 1, 2, 2)
; hanoi(0, 2, 1, 1)
0 2 ; movedisk
0 1 ; movedisk
; hanoi(2, 1, 0, 1)
2 1 ; movedisk
0 2 ; movedisk ***
; hanoi(1, 2, 0, 2)
; hanoi(1, 0, 2, 1)
1 0 ; movedisk
1 2 ; movedisk
; hanoi(0, 2, 1, 1)
0 2 ; movedisk
Because of the recursive nature of the solution, the FROM and TO columns follow a recursive logic: if you take the middle entry on the columns, the parts above and below are copies of each others, but with the numbers permuted. This is an obvious consequence of the algorithm itself which does not perform any arithmetics on the peg numbers but only permutes them. In the case N=4 the middle row is at x=4 (marked with three stars above).
Now the expression (X & (X-1)) unsets the least set bit of X, so it maps e.g. numbers form 1 to 7 like this:
1 -> 0
2 -> 0
3 -> 2
4 -> 0 (***)
5 -> 4 % 3 = 1
6 -> 4 % 3 = 1
7 -> 6 % 3 = 0
The trick is that because the middle row is always at an exact power of two and thus has exactly one bit set, the part after the middle row equals the part before it when you add the middle row value (4 in this case) to the rows (i.e. 4=0+4, 6=2+6). This implements the "copy" property, the addition of the middle row implements the permutation part. The expression (X | (X-1)) + 1 sets the lowest zero bit which has ones to its right, and clears these ones, so it has similar properties as expected:
1 -> 2
2 -> 4 % 3 = 1
3 -> 4 % 3 = 1
4 -> 8 (***) % 3 = 2
5 -> 6 % 3 = 0
6 -> 8 % 3 = 2
7 -> 8 % 3 = 2
As to why these sequences actually produce the correct peg numbers, let's consider the FROM column. The recursive solution starts with hanoi(0, 2, 1, N), so at the middle row (2 ** (N-1)) you must have movedisk(0, 2). Now by the recursion rule, at (2 ** (N-2)) you need to have movedisk(0, 1) and at (2 ** (N-1)) + 2 ** (N-2) movedisk (1, 2). This creates the "0,0,1" pattern for the from pegs which is visible with different permutations in the table above (check rows 2, 4 and 6 for 0,0,1 and rows 1, 2, 3 for 0,0,2, and rows 5, 6, 7 for 1,1,0, all permuted versions of the same pattern).
Now then of all the functions that have this property that they create copies of themselves around powers of two but with offsets, the authors have selected those that produce modulo 3 the correct permutations. This isn't an overtly difficult task because there are only 6 possible permutations of the three integers 0..2 and the permutations progress in a logical order in the algorithm. (X|(X-1))+1 is not necessarily deeply linked with the Hanoi problem or it doesn't need to be; it's enough that it has the copying property and that it happens to produce the correct permutations in the correct order.
antti.huima's solution is essentially correct, but I wanted something more rigorous, and it was too big to fit in a comment. Here goes:
First note: at the middle step x = 2N-1 of this algorithm, the "from" peg is 0, and the "to" peg is 2N % 3. This leaves 2(N-1) % 3 for the "spare" peg.
This is also true for the last step of the algorithm, so we see that actually the authors' algorithm
is a slight "cheat": they're moving the disks from peg 0 to peg 2N % 3, rather than a fixed,
pre-specified "to" peg. This could be changed with not much work.
The original Hanoi algorithm is:
hanoi(from, to, spare, N):
hanoi(from, spare, to, N-1)
move(from, to)
hanoi(spare, to, from, N-1)
Plugging in "from" = 0, "to" = 2N % 3, "spare" = 2N-1 % 3, we get (suppressing the %3's):
hanoi(0, 2**N, 2**(N-1), N):
(a) hanoi(0, 2**(N-1), 2**N, N-1)
(b) move(0, 2**N)
(c) hanoi(2**(N-1), 2**N, 0, N-1)
The fundamental observation here is:
In line (c), the pegs are exactly the pegs of hanoi(0, 2N-1, 2N, N-1) shifted by 2N-1 % 3, i.e.
they are exactly the pegs of line (a) with this amount added to them.
I claim that it follows that when we
run line (c), the "from" and "to" pegs are the corresponding pegs of line (a) shifted by 2N-1 % 3. This
follows from the easy, more general lemma that in hanoi(a+x, b+x, c+x, N), the "from and "to" pegs are shifted exactly x from in hanoi(a, b, c, N).
Now consider the functions
f(x) = (x & (x-1)) % 3
g(x) = (x | (x-1)) + 1 % 3
To prove that the given algorithm works, we only have to show that:
f(2N-1) == 0 and g(2N-1) == 2N
for 0 < i < 2N, we have f(2N - i) == f(2N + i) + 2N % 3, and g(2N - i) == g(2N + i) + 2N % 3.
Both of these are easy to show.
This isn't directly answering the question, but it was too long to put in a comment.
I had always done this by analyzing the size of disk you should move next. If you look at the disks moved, it comes out to:
1 disk : 1
2 disks : 1 2 1
3 disks : 1 2 1 3 1 2 1
4 disks : 1 2 1 3 1 2 1 4 1 2 1 3 1 2 1
Odd sizes always move in the opposite direction of even ones, in order if pegs (0, 1, 2, repeat) or (2, 1, 0, repeat).
If you take a look at the pattern, the ring to move is the highest bit set of the xor of the number of moves and the number of moves + 1.

Resources